CMA Study Group

 View Only
  • 1.  CMA PART 1 QUESTION

    Posted 10-05-2021 03:52 PM
    Hi Guys,

    I came across this question and I need help understanding it, if anyone understands it I would appreciate an explanation, thanks:

    The following forecasted information is available for a manufacturing division for next year:
    Category
    Amount
    (thousands)
    Working capital
    $  1,800
    Revenue
      30,000
    Plant and equipment
      17,200
    To establish a standard of performance for the division's manager using the residual income approach, four scenarios are being considered.
    Imputed Interest
    Target
    Residual Income
    1
    15%
    $2,000,000
    2
    12%
      1,500,000
    3
    18%
      1,250,000
    4
    10%
      2,500,000
    Which scenario assumes the lowest maximum cost?
    Answer (D) is correct.
    Residual income is the excess of the amount of the ROI over a targeted amount equal to an imputed interest charge on invested capital. If a manager has $19,000,000 of invested capital ($17,200,000 of plant and equipment + $1,800,000 of working capital), a 15% imputed interest charge equals $2,850,000. Adding $2,000,000 of residual income to the imputed interest results in a target profit of $4,850,000. This profit can be achieved if costs are $25,150,000 ($30,000,000 revenue – $4,850,000 profit).


    ------------------------------
    Tiffany Borello
    Other
    New York NY
    United States
    ------------------------------


  • 2.  RE: CMA PART 1 QUESTION

    Posted 10-06-2021 02:19 AM
    Hi Tiffany,

    the question here asks whats the maximum costs the company can incur to result ROI of given options.The formula for ROI = Operating Income - Opportunity cost. Opportunity cost here is (Operating assets x imputed rate of return)
    with option D (scenario one)
    ROI = $2,000,000 (given)
    Opportunity costs = 2,850,000 ( 17,200,000 + 1,800,000 x 15%)
    Operating income = $30,000,000 (revenue) - X ( costs) which we have to arrive
    A bit of algebra will be helpful here

    30,000,000 - x - 2,850,000 = 2,000,000
    30,000,000 - x = 2,000,000 + 2,850,000 = 4,850,000
    -x = 4850,000 - 30,000,000 = -25,150,000
    -x = -25,150,000, minus gets cancelled and the X = 25,150,000 which is the maximum cost incurred to get the stated ROI in the given question. You can do the same for all the other option to find out the maximum cost. In this case, its option D with costs being $25,150,000. Hope this helps.

    ------------------------------
    Rajnivas Badrinarayannan
    Chennai TN
    India
    ------------------------------



  • 3.  RE: CMA PART 1 QUESTION

    Posted 10-06-2021 09:08 AM
    Yes this helped, thank you!

    ------------------------------
    Tiffany Borello
    Other
    New York NY
    United States
    ------------------------------



  • 4.  RE: CMA PART 1 QUESTION

    Posted 10-07-2021 06:28 AM
    ROI=income of business unit/assets of business unit
    Residual income = income of business unit - (assets of business unit x required rate of return)

    ------------------------------
    GEZOULI ELTAYEB ALI
    Accountant
    Riyadh
    Saudi Arabia
    ------------------------------



  • 5.  RE: CMA PART 1 QUESTION

    Posted 10-07-2021 06:30 AM
    ROI=income of business unit/assets of business unit
    Residual income = income of business unit - (assets of business unit x required rate of return)

    ------------------------------
    GEZOULI ELTAYEB ALI
    Accountant
    Riyadh
    Saudi Arabia
    ------------------------------